Quiz 1 Flashcards
Concern is raised in an 86-year-old male with non-insulin-dependent diabetes mellitus (NIDDM), or type II diabetes, about the possibility of hypoglycemia when considering the use of an oral hypoglycemic agent. Which of the following antidiabetic drugs is least likely to cause hypoglycemia? A. Metformin B. Chlorpropamide C. Insulin D. Glyburide E. All of the above
A. Metformin
Pretest
A woman goes into premature labor early enough that there are great concerns about inadequate fetal lung development and the risk of fetal respiratory distress syndrome. Ritodrine therapy is started to slow labor, but parturition seems imminent. Which of the following adjuncts should be administered prepartum, specifically for the purpose of reducing the risks and complications of the newborn's immature respiratory system development? a. Albuterol (beta2 agonist) b. Betamethasone c. Ergonovine (or methylergonovine) d. Indomethacin e. Magnesium sulfate
b. Betamethasone
Pretest
A 23-year-old woman with asthma has what is described as “aspirin (hyper)sensitivity” and experiences severe bronchospasm in response to even small doses of the drug. Which of the following is the most likely mechanism by which the aspirin provokes her pulmonary problems?
a. Blocks synthesis of endogenous prostaglandins that have bronchodilator activity
b. Induces formation of antibodies directed against the salicylate on airway mast cells
c. Induces hypersensitivity of H1 receptors on airway smooth muscles
d. Induces hypersensitivity of muscarinic receptors on airway smooth muscles
e. Prevents or reduces epinephrine binding to beta-2-adrenergic receptors (airways and elsewhere)
a. Blocks synthesis of endogenous prostaglandins that have bronchodilator activity
Pretest
A 70-year-old man has a history of ulcer disease. He has recently experienced swelling and pain in the joints of his hands. His physician wants to begin therapy with an NSAID. Which one of the follow- ing drugs might also be prescribed along with the NSAID to reduce the risk of activating this patient’s ulcer disease? A. Allopurinol. B. Colchicine. C. Misoprostol. D. Probenecid. E. Sulindac.
C. Misoprostol.
Lippincott
A 12-year-old girl with a childhood history of asthma complained of cough, dyspnea, and wheezing after visiting a riding stable. Her symptoms became so severe that her parents brought her to the emergency room. Physical examination revealed diaphoresis, dyspnea, tachycardia, and tachypnea. Her respira- tory rate was 42 breaths per minute, pulse rate 110 beats per minute, and blood pressure 132/65 mm Hg. Which of the following is the most appropriate drug to rapidly reverse her bronchoconstriction? A. Inhaled fluticasone. B. Inhaled beclomethasone. C. Inhaled albuterol. D. Intravenous propranolol. E. Oral theophylline.
C. Inhaled albuterol.
Lippincott
A patient has acute gout. The physician initially thinks about prescribing just one or two oral doses of colchicine, 12 h apart, but then decides otherwise. The main reason for avoiding colchicine, even with a very short oral course, is the development of which of the following?
a. Bone marrow suppression
b. Bronchospasm
c. GI distress that is almost as bad as the acute gout discomfort
d. Hepatotoxicity
e. One or two oral doses seldom relieve gout pain
f. Refractoriness/tolerance with just a dose or two
c. “GI distress” that is almost as bad as the acute gout discomfort
pretest
A woman has severe, irritable bowel syndrome characterized by fre- quent, profuse, and symptomatic diarrhea. She has not responded to first- line therapies and is started on alosetron. Which of the following is the most worrisome adverse effect associated with this drug?
a. Cardiac arrhythmias (serious, e.g., ventricular fibrillation)
b. Constipation, bowel impaction, ischemic colitis
c. Parkinsonian extrapyramidal reactions
d. Pulmonary fibrosis
e. Renal failure
b. Constipation, bowel impaction, ischemic colitis
Pretest
A 43-year-old woman becomes hypertensive and suffers a fatal acute
coronary syndrome shortly after starting therapy on a drug. Autopsy shows little in the way of coronary atherosclerosis, but EKG changes noted just before her death revealed significant myocardial ischemia in the
myocardium served by the left anterior descending and circumflex coronary arteries. The cause of death is thought to involve coronary vasospasm. Which of the following drugs most likely precipitated this event?
a. Bromocriptine for Parkinson’s disease
b. Ergotamine given to abort a migraine attack
c. Morphine for post-trauma analgesia
d. Phenoxybenzamine used for carcinoid syndrome
e. Phenytoin to manage generalized tonic-clonic seizures
b. Ergotamine given to abort a migraine attack
Pretest
a patient with RA is treated with ibuprofen, but joint pain and stiffness are increasing. His physician prescribes another drug that may slow the progression of the disease that is to be used with ibuprofen. Unfortunately, side effects develop, including dizzines, tinnitus, blurred vision, halos around bright lights, and pruritis. Ocular examination reveals corneal deposits and slight retinal pigmentation. The drug more recently prescribed is likely to be: A. Auranofin or gold salts B. Hydroxychloroquine C. Etanercept D. Methotrexate E. Thioridazine
B. Hydroxychloroquine
USMLE Step 1 Lecture Notes
a 58 year old woman comes to the clinic because of a 3-week history of pain in the area of her eye, blurred vision, nausea, and vomiting. A tonometer shows an increase in intraocular pressure. Prostaglandin analog (latanoprost) is initiated. Which of the following best describes the method of action of this drug in this patient?
A. Contraction of the ciliary muscle
B. Inhibition of carbonic anhydrase
C. Increased uveoslceral outflow of aqueous humor
D. Increased production of aqueous humor
E. Decreased production of aqueous humor
C. Increased uveoslceral outflow of aqueous humor
Your patient has rheumatoid arthritis that has been refractory to diclofenac, ibuprofen, indomethacin, and sulindac. In addition, she has experienced numerous GI bleeds in response to those drugs. We start her on therapy with etanercept. Which of the following is the most likely mechanism by which etanercept suppresses the signs, symptoms, or underlying pathophysiology of rheumatoid arthritis?
a. Inhibits eicosanoid synthesis by inhibiting phospholipase A2
b. Inhibits leukocyte migration by blocking microtubular formation
c. Neutralizes circulating tumor necrosis factor (α-TNF)
d. Selectively and effectively inhibits COX-2
e. Stimulates collagen and mucopolysaccharide synthesis in the joints
c. Neutralizes circulating tumor necrosis factor (α-TNF)
A patient has been taking one of the drugs listed below for about 4 months and is experiencing the desired therapeutic effects from it. The MD now prescribes indomethacin to treat a particularly severe flare-up of rheuma- toid arthritis. Within a matter of days the therapeutic effects of the first drug wane dramatically, its actions antagonized by the indomethacin. Which of the following was the most likely drug affected by the indomethacin?
a. Allopurinol, given for prophylaxis of hyperuricemia
b. Captopril, given for essential hypertension
c. Fexofenadine, given for managing seasonal allergy responses
d. Sumatriptan, given for abortive therapy of migraine headaches
e. Warfarin, given for prophylaxis of venous thrombosis
b. Captopril, given for essential hypertension
a 76 year old woman with a history of uncontrolled hypertension presents to the emergency department with hypotension and shock-like symptoms. Her daughter reported systolic blood pressures near 200 mmHg earlier in the day, and suspects a dissecting aneurysm, which is confirmed with a CT of the chest. Biopsy of the repaired aorta shows giant cell arteritis, and the woman is placed on a regimen of high dose prednisone. The anti-inflammatory effect of exogenous glucocorticoids is though to be due to which of the following?
A. Increased capillary membrane permeability
B. Increased release of interleukin-1 (pyrogen) from granulocytes
C. Inhibition of Phospholipase A2
D. Activation of Phospholipase A2
E. Increased formation of leukotrienes
C. Inhibition of Phospholipase A2
Physio Pretest
A 4 year old girl who is small for her age is diagnosed with juvenile rheumatoid arthritis. Her liver and spleen are palpable, and her joints are swollen and tender. She is unable to move around as freely as she did before. Which of the following is the initial treatment of choice for this patient? A. Prednisone B. Acetaminophen C. Methotrexate D. Naproxen E. Etanercept
D. Naproxen
A patient suffering status asthmaticus presents in the emergency department. Blood gases reveal severe respiratory acidosis and hypoxia. Even large parenteral doses of a selective β2 agonist fail to dilate the airways adequately; rather, they cause dangerous degrees of tachycardia. Which of the following pharmacologic interventions or approaches is most likely to control the acute symptoms and restore the bronchodilator efficacy of the adrenergic drug?
a. Add inhaled cromolyn
b. Give a parenteral corticosteroid
c. Give parenteral diphenhydramine
d. Switch to epinephrine
e. Switch to isoproterenol (β1/β2 agonist)
b. Give a parenteral corticosteroid